Combined CH 17 PREP U

Pataasin ang iyong marka sa homework at exams ngayon gamit ang Quizwiz!

The nurse is teaching a client who is diagnosed with preeclampsia how to monitor her condition. The nurse determines the client needs more instruction after making which statement?

"If I have changes in my vision, I will lie down and rest."

The cerclage, or purse string suture is inserted into the cervix to prevent preterm cervical dilatation and pregnancy loss. Staples, glue, or a cervical cap will not prevent the cervix from dilating.

"Purse-string sutures are placed in the cervix to prevent it from dilating."

A student nurse asks the instructor what percentage of clinically recognized pregnancies end in miscarriages during the first trimester. Which response from the nurse is the most accurate?

15% to 20%

The nurse is identifying nursing diagnoses for a patient with gestational hypertension. Which diagnosis would be the most appropriate for this patient?

Ineffective tissue perfusion related to vasoconstriction of blood vessels

When assessing a pregnant woman with vaginal bleeding, the nurse would suspect a threatened abortion based on which finding?

slight vaginal bleeding

The nurse is required to assess a pregnant client who is reporting vaginal bleeding. Which nursing action is the priority?

assessing the amount and color of the bleeding

The nurse is caring for a client who has a multifetal pregnancy. What topic should the nurse prioritize during health education?

Signs of preterm labor The client with a multifetal pregnancy must be made aware of the risks posed by preterm labor. There is no corresponding increase in the risk for hypertension or blood incompatabilities. Parenting skills are secondary to physiologic needs at this point.

A client is 20 weeks pregnant. At a prenatal visit, the nurse begins the prenatal assessment. Which finding would necessitate calling the primary care provider to assess the client?

The client has pink vaginal discharge and pelvic pressure. Cervical dilatation usually occurs painlessly, and often the first symptom is pink vaginal discharge or increased pelvic pressure, which then is followed by rupture of membranes and discharge of the amniotic fluid. The other answers are nonthreatening signs and symptoms.

A client has come to the office for a prenatal visit during her 22nd week of gestation. On examination, it is noted that her blood pressure has increased to 138/90 mm Hg. Her urine is negative for proteinuria. The nurse recognizes which factor as the potential cause?

gestational hypertension

A novice nurse asks to be assigned to the least complex antepartum client. Which condition would necessitate the least complex care requirements?

gestational hypertension

A pregnant woman has arrived to the office reporting vaginal bleeding. Which finding during the assessment would lead the nurse to suspect an inevitable abortion?

strong abdominal cramping

The nurse is caring for a pregnant client with severe preeclampsia. Which nursing intervention should a nurse perform to institute and maintain seizure precautions in this client?

Keep the suction equipment readily available. The nurse should institute and maintain seizure precautions such as padding the side rails and having oxygen, suction equipment, and call light readily available to protect the client from injury. The nurse should provide a quiet, darkened room to stabilize the client. The nurse should maintain the client on complete bed rest in the left lateral lying position and not in a supine position. Keeping the head of the bed slightly elevated will not help maintain seizure precautions.

A woman in labor is at risk for abruptio placentae. Which assessment would most likely lead the nurse to suspect that this has happened?

sharp fundal pain and discomfort between contractions

After a regular prenatal visit, a pregnant client asks the nurse to describe the differences between abruptio placentae and placenta previa. Which statement should the nurse include in the teaching?

"Placenta previa causes painless, bright red bleeding during pregnancy due to an abnormally implanted placenta that is too close to or covers the cervix; abruptio placenta is associated with dark red painful bleeding caused by premature separation of the placenta from the wall of the uterus before the end of labor."

The nurse is identifying nursing diagnoses for a patient with gestational hypertension. Which diagnosis would be the most appropriate for this patient?

Ineffective tissue perfusion related to vasoconstriction of blood vessels In gestational hypertension, vasospasm occurs in both small and large arteries during pregnancy. This can lead to ineffective tissue perfusion. There is no evidence to suggest that the fetus is in distress. There is no enough information to support imbalanced nutrition. Gestational hypertension does not affect heart contractions.

A client is diagnosed with gestational hypertension and is receiving magnesium sulfate. Which finding would the nurse interpret as indicating a therapeutic level of medication? urinary output of 20 mL per hour difficulty in arousing deep tendons reflexes 2+ respiratory rate of 10 breaths/minute

deep tendons reflexes 2+

Which is the best question the nurse can ask a woman who is leaving the hospital after experiencing a complete spontaneous abortion?

"Do you have someone to talk to, or may I give you the names and numbers for some possible grief counselors?" When a woman has a spontaneous abortion one important consideration is the emotional needs of the woman once she is home. She may not want to talk about the loss for a period of time, but the nurse needs to determine her support system for the future. Asking the woman if she is "going to try again" is an inappropriate question for the nurse to ask and diminishes the experience of having a spontaneous abortion. Giving the woman statistical information on spontaneous abortions is not appropriate when this client needs support and caring concern. Offering to give the client resources to aid in smoking cessation is not addressed in the scenario, so this is an inappropriate response.

A pregnant client at 32 weeks' gestation calls the clinic and informs the nurse that she thinks her membranes are leaking. She states that some clear fluid has run down her leg. What is the best response by the nurse?

"It is best for you to visit a hospital immediately. They can use a nitrazine strip to determine if it is amniotic fluid." The practitioner will perform a speculum examination, looking for pooling of amniotic fluid, and then test the fluid with nitrazine paper, which turns blue in the presence of amniotic fluid. Preterm premature rupture of membranes occurs when the rupture of the amniotic sac before the onset of labor happens in a woman who is less than 37 weeks' gestation.

A client with a history of cervical insufficiency is seen for reports of pink-tinged discharge and pelvic pressure. The primary care provider decides to perform a cervical cerclage. The nurse teaches the client about the procedure. Which client response indicates that the teaching has been effective?

"Purse-string sutures are placed in the cervix to prevent it from dilating."

A client with a history of cervical insufficiency is seen for reports of pink-tinged discharge and pelvic pressure. The primary care provider decides to perform a cervical cerclage. The nurse teaches the client about the procedure. Which client response indicates that the teaching has been effective?

"Purse-string sutures are placed in the cervix to prevent it from dilating." The classic manifestations of abruption placenta are painful dark red vaginal bleeding, "knife-like" abdominal pain, uterine tenderness, contractions, and decreased fetal movement. Painless bright red vaginal bleeding is the clinical manifestation of placenta previa. Generalized vasospasm is the clinical manifestation of preeclampsia and not of abruptio placenta.

A client suffering a miscarriage at 12 weeks' gestations is very upset that the health care provider has ordered a D&C. How should the nurse respond after the client states she didn't have a D&C the time she lost a previous baby at 5 weeks' gestation?

"This procedure is needed to adequately remove all the fetal tissue."

The nurse is assisting a client who has just undergone an amniocentesis. Blood results indicate the mother has type O blood and the fetus has type AB blood. The nurse should point out the mother and fetus are at an increased risk for which situation related to this procedure?

Baby developing postbirth jaundice

A patient is admitted with a diagnosis of ectopic pregnancy. For what should the nurse anticipate preparing the patient?

Immediate surgery

A pregnant woman has been admitted to the hospital due to severe preeclampsia. Which measure will be important for the nurse to include in the care plan?

Institute and maintain seizure precautions.

The woman with severe preeclampsia should be maintained on complete bed rest in a dark and quiet room to avoid stimulation. The client is at risk for seizures; therefore, institution and maintenance of seizure precautions should be in place.

Institute and maintain seizure precautions.

The nurse is caring for a pregnant client with fallopian tube rupture. Which intervention is the priority for this client?

Monitor the client's vital signs and bleeding.

A client at 37 weeks' gestation presents to the emergency department with a BP 150/108 mm Hg, 1+ pedal edema, 1+ proteinuria, and normal deep tendon reflexes. Which assessment should the nurse prioritize as the client is administered magnesium sulfate IV?

Respiratory rate

The nurse is caring for a client who has a multifetal pregnancy. What topic should the nurse prioritize during health education?

Signs of preterm labor

A nurse is monitoring a client with PROM who is in labor and observes meconium in the amniotic fluid. What does the observation of meconium indicate?

fetal distress related to hypoxia

A client has been admitted with abruptio placentae. She has lost 1,200 mL of blood, is normotensive, and ultrasound indicates approximately 30% separation. The nurse documents this as which classification of abruptio placentae?

grade 2

A woman is admitted with a diagnosis of ectopic pregnancy. For which action would the nurse anticipate beginning preparation?

immediate surgery Ectopic pregnancy means an embryo has implanted outside the uterus, usually in the fallopian tube. Surgery is usually necessary to remove the growing structure before the tube ruptures or repair the tube if rupture has already occurred. Bed rest will not correct the problem of an ectopic pregnancy. Administering a tocolytic is not indicated, nor is internal uterine monitoring.

What would be the physiologic basis for a placenta previa?

low placental implantation The cause of placenta previa is usually unknown, but for some reason the placenta is implanted low instead of high on the uterus.

A client in her 20th week of gestation develops HELLP syndrome. What are features of HELLP syndrome? Select all that apply.

low platelet count elevated liver enzymes hemolysis The HELLP syndrome is a syndrome involving hemolysis (microangiopathic hemolytic anemia), elevated liver enzymes, and a low platelet count. Hyperthermia and leukocytosis are not features of HELLP syndrome.

After an examination, a client has been determined to have an unruptured ectopic pregnancy. Which medication would the nurse anticipate being prescribed?

methotrexate

A 24-year-old client presents in labor. The nurse notes there is an order to administer RhoGAM after the birth of her infant. When asked by the client the reason for this injection, which reason should the nurse point out?

prevent maternal D antibody formation. Because RhoGAM contains passive antibodies, the solution will prevent the woman from forming long-lasting antibodies which may harm a future fetus. The administration of RhoGAM does not promote the formation of maternal D antibodies; it does not stimulate maternal D immune antigens or prevent fetal Rh blood formation.

The nurse is caring for a client with preeclampsia and understands the need to auscultate this client's lung sounds every two hours. Why would the nurse do this?

pulmonary edema In the hospital, monitor blood pressure at least every four hours for mild preeclampsia and more frequently for severe disease. In addition, it is important to auscultate the lungs every two hours. Adventitious sounds may indicate developing pulmonary edema.

When assessing a pregnant woman with vaginal bleeding, the nurse would suspect a threatened abortion based on which finding?

slight vaginal bleeding Slight vaginal bleeding early in pregnancy, no cervical dilation, and a closed cervical os are associated with a threatened abortion. Strong abdominal cramping is associated with an inevitable abortion. With an inevitable abortion, passage of the products of conception may occur. No fetal tissue is passed with a threatened abortion.

A young mother gives birth to twin boys who shared the same placenta. What serious complication are they at risk for?

twin-to-twin transfusion syndrome (TTTS)

The nurse is preparing discharge instructions for a pregnant patient experiencing preterm rupture of membranes. What should the nurse include in this teaching? Select all that apply.

1. Avoid douching. 2. Measure oral temperature twice a day.

A nurse is assessing a pregnant client with preeclampsia for suspected dependent edema. Which description of dependent edema is most accurate?

Dependent edema may be seen in the sacral area if the client is on bed rest.

A pregnant client with hyperemesis gravidarum needs advice on how to minimize nausea and vomiting. Which instruction should the nurse give this client?

Eat small, frequent meals throughout the day. The nurse should instruct the client with hyperemesis gravidarum to eat small, frequent meals throughout the day to minimize nausea and vomiting. The nurse should also instruct the client to avoid lying down or reclining for at least 2 hours after eating and to increase the intake of carbonated beverages. The nurse should instruct the client to try foods that settle the stomach such as dry crackers, toast, or soda.

A patient recovering from an uneventful vaginal delivery is prescribed Rho(D) immune globulin (RhIG). What should the nurse explain to the patient regarding the purpose of this medication?

It prevents maternal D antibody formation. Rho(D) immune globulin (RhIG) is given to Rh-negative pregnant patients to prevent the formation of maternal antibodies to the Rh-positive blood type of the developing fetus. This medication does not prevent fetal Rh blood formation, stimulate maternal immune antigens, or promote maternal antibody formation.

A client at 27 weeks' gestation is admitted to the OB unit afer reporting headaches and edema of her hands. Review of the prenatal notes reveals BP consistently above 136/90 mm Hg. The nurse anticipates the health care provider will order magneisum sulfate to accomplish which primary goal?

Prevent maternal seizures

A pregnant woman has arrived to the office reporting vaginal bleeding. Which finding during the assessment would lead the nurse to suspect an inevitable abortion?

Strong abdominal cramping

A primipara at 36 weeks' gestation is being monitored in the prenatal clinic for risk of preeclampsia. Which sign or symptom is the priority concern for the nurse?

a dipstick value of 2+ for protein The increasing amount of protein in the urine is a concern the preeclampsia may be progressing to severe preeclampsia. The woman needs further assessment by the health care provider. Dependent edema may be seen in the majority of pregnant women and is not an indicator of progression from preeclampsia to eclampsia. Weight gain is no longer considered an indicator for the progression of preeclampsia. A systolic blood pressure increase is not the highest priority concern for the nurse, since there is no indication what the baseline blood pressure was

A woman is being closely monitored and treated for severe preeclampsia with magnesium sulfate. Which finding would alert the nurse to the development of magnesium toxicity in this client?

diminished reflexes Diminished or absent reflexes occur when a client develops magnesium toxicity. Elevated liver enzymes are unrelated to magnesium toxicity and may indicate the development of HELLP syndrome. The onset of seizure activity indicates eclampsia. A serum magnesium level of 6.5 mEq/L would fall within the therapeutic range of 4 to 7 mEq/L.

A pregnant client has been admitted with reports of brownish vaginal bleeding. On examination there is an elevated hCG level, absent fetal heart sounds, and a discrepancy between the uterine size and the gestational age. The nurse interprets these findings to suggest which condition?

gestational trophoblastic disease The client is most likely experiencing gestational trophoblastic disease or a molar pregnancy. In gestational trophoblastic disease or molar pregnancy, there is an abnormal proliferation and eventual degeneration of the trophoblastic villi. The signs and symptoms of molar pregnancy include brownish vaginal bleeding, elevated hCG levels, discrepancy between the uterine size and the gestational age, and absent fetal heart sounds. Abruption of placenta is characterized by premature separation of the placenta. Ectopic pregnancy is a condition where there is implantation of the blastocyst outside the uterus. In placenta previa the placental attachment is at the lower uterine segment.

A client in her 20th week of gestation develops HELLP syndrome. What are features of HELLP syndrome? Select all that apply.

hemolysis elevated liver enzymes low platelet count The HELLP syndrome is a syndrome involving hemolysis (microangiopathic hemolytic anemia), elevated liver enzymes, and a low platelet count. Hyperthermia and leukocytosis are not features of HELLP syndrome

A woman in labor is at risk for abruptio placentae. Which assessment would most likely lead the nurse to suspect that this has happened?

sharp fundal pain and discomfort between contractions An abruptio placentae refers to premature separation of the placenta from the uterus. As the placenta loosens, it causes sharp pain. Labor begins with a continuing nagging sensation.

A client at 11 weeks' gestation experiences pregnancy loss. The client asks the nurse if the bleeding and cramping that occurred during the miscarriage were caused by working long hours in a stressful environment. What is the most appropriate response from the nurse?

"I can understand your need to find an answer to what caused this. Let's talk about this further." Talking with the client may assist her to explore her feelings. She and her family may search for a cause for a spontaneous early bleeding so they can plan for future pregnancies. Even with modern technology and medical advances, however, a direct cause cannot usually be determined.

A 16-year-old client gave birth to a 12 weeks' gestation fetus last week. The client has come to the office for follow-up and while waiting in an examination room notices that on the schedule is written her name and "follow-up of spontaneous abortion." The client is upset about what is written on the schedule. How can the nurse best explain this terminology?

"Spontaneous abortion is a more specific term used to describe a spontaneous miscarriage, which is a loss of pregnancy before 20 weeks. This term does not imply that you did anything to affect the pregnancy."

A nurse has been assigned to assess a pregnant client for abruptio placenta. For which classic manifestation of this condition should the nurse assess?

"knife-like" abdominal pain with vaginal bleeding

The nurse is preparing the plan of care for a woman hospitalized for hyperemesis gravidarum. Which interventions would the nurse most likely include? Select all that apply.

1. maintaining NPO status for the first day or two 2. administering antiemetic agents 3. obtaining baseline blood electrolyte levels 4. monitoring intake and output

A pregnant client at 32 weeks' gestation is treated with magnesium sulfate for seizure management. The nurse assesses which of the following for evidence of magnesium toxicity? Absence of knee jerk response Increased blood pressure Increased rate of respiration Frequency of micturition

Absence of knee jerk response Explanation: Magnesium sulfate toxicity is characterized by absence of deep tendon reflexes like the knee jerk reflex. Urinary retention, and not frequency of micturition, is seen with magnesium sulfate toxicity. Magnesium sulfate is given to treat seizures associated with hypertension and proteinuria in pregnancy, and therefore decreases the blood pressure. It does not cause an increase in blood pressure. There is respiratory depression, and not an increased rate of respiration, with magnesium sulfate toxicity.

A nurse is caring for a client with hyperemesis gravidarum. Which nursing action is the priority for this client?

Administer IV NS with vitamins and electrolytes. The first choice for fluid replacement is generally NS with vitamins and electrolytes added. If the client does not improve after several days of bed rest, "gut rest," IV fluids, and antiemetics, then total parenteral nutrition or percutaneous endoscopic gastrostomy tube feeding is instituted to prevent malnutrition.

A primigravida 28-year-old client is noted to have Rh negative blood and her husband is noted to be Rh positive. The nurse should prepare to administer RhoGAM after which diagnostic procedure?

Amniocentesis

A client has been admitted to the hospital with a diagnosis of severe preeclampsia. Which nursing intervention is the priority?

Confine the client to bed rest in a darkened room.

A client tells that nurse in the doctor's office that her friend developed high blood pressure on her last pregnancy. She is concerned that she will have the same problem. What is the standard of care for preeclampsia? Monitor the client for headaches or swelling on the body. Have her blood pressure checked at every prenatal visit. Take low-dose antihypertensive prophylactically. Take one aspirin every day.

Have her blood pressure checked at every prenatal visit. Explanation: Preeclampsia and eclampsia are common problems for pregnant clients and require regular blood pressure monitoring at all prenatal visits. Antihypertensives are not prescribed unless the client is already hypertensive. Monitoring for headaches and swelling is a good predictor of a problem but doesn't address prevention— nor does it predict who will have hypertension. Taking aspirin has shown to reduce the risk in women who have moderate to high risk factors, but has shown no effect on those women with low risk factors.

A patient recovering from an uneventful vaginal delivery is prescribed Rho(D) immune globulin (RhIG). What should the nurse explain to the patient regarding the purpose of this medication? It stimulates maternal D immune antigens. It promotes maternal D antibody formation. It prevents fetal Rh blood formation. It prevents maternal D antibody formation.

It prevents maternal D antibody formation. Explanation: Rho(D) immune globulin (RhIG) is given to Rh-negative pregnant patients to prevent the formation of maternal antibodies to the Rh-positive blood type of the developing fetus. This medication does not prevent fetal Rh blood formation, stimulate maternal immune antigens, or promote maternal antibody formation.

A 32-year-old gravida 3 para 2 at 36 weeks' gestation comes to the obstetric department reporting abdominal pain. Her blood pressure is 164/90 mm/Hg, her pulse is 100 beats per minute, and her respirations are 24 per minute. She is restless and slightly diaphoretic with a small amount of dark red vaginal bleeding. What assessment should the nurse make next?

Palpate the fundus, and check fetal heart rate. The classic signs of abruptio placentae are pain, dark red vaginal bleeding, a rigid, board-like abdomen, hypertonic labor, and fetal distress.

A pregnant patient is developing HELLP syndrome. During labor, which order should the nurse question?

Prepare for epidural anesthesia.

The following hourly assessments are obtained by the nurse on a client with preeclampsia receiving magnesium sulfate: 97.3oF (36.2oC), HR 88, RR 1, BP 148/110 mm Hg. What other priority physical assessments by the nurse should be implemented to assess for potential toxicity?

Reflexes

A 24-year-old client is brought to the emergency department complaining of severe abodminal pain, vaginal bleeding, and fatigue. The nurse notes on assessment cool, clammy skin, confusion, and vital signs: HR 130, RR 28, and BP 98/60 mm Hg. Which action should the nurse prioritize?

Rule out shock.

A nurse is describing the use of Rho(D) immune globulin as the therapy of choice for isoimmunization in Rh-negative women and for other conditions to a group of nurses working at the women's health clinic. The nurse determines that additional teaching is needed when the group identifies which situation as an indication for Rho(D) immune globulin?

STIs

The nurse is monitoring a pregnant patient who is receiving intravenous magnesium sulfate for eclampsia. During the last assessment, the nurse was unable to elicit a patellar reflex. What should the nurse do?

Stop the current infusion.

A client is 20 weeks pregnant. At a prenatal visit, the nurse begins the prenatal assessment. Which finding would necessitate calling the primary care provider to assess the client?

The client has pink vaginal discharge and pelvic pressure.

Vaginal bleeding during pregnancy is always a deviation from the normal.

True Vaginal bleeding during pregnancy is always a deviation from the normal, is always potentially serious, may occur at any point during pregnancy, and is always frightening. It must always be carefully investigated because it can impair both the outcome of the pregnancy and the woman's life or health.

A pregnant woman is admitted to the hospital with a diagnosis of placenta previa. Which action would be the priority for this woman on admission? assessing fetal heart tones by use of an external monitor performing a vaginal examination to assess the extent of bleeding helping the woman remain ambulatory to reduce bleeding assessing uterine contractions by an internal pressure gauge

assessing fetal heart tones by use of an external monitor Explanation: Not disrupting the placenta is a prime responsibility. An internal monitor, a vaginal examination, and remaining ambulatory could all do this and thus are contraindicated.

When providing counseling on early pregnancy loss, the nurse should discuss which factor as the most common cause for spontaneous abortion?

chromosomal abnormality The most common cause for the loss of a fetus in the first trimester is associated with a genetic defect or chromosomal abnormality. There is nothing that can be done, and the mother should feel no fault. The nurse needs to educate the parents to speak with a health care provider for further information and questions related to genetic testing. Early pregnancy loss is not associated with maternal smoking, lack of prenatal care, or the age of the mother.

Diminished or absent reflexes occur when a client develops magnesium toxicity. Elevated liver enzymes are unrelated to magnesium toxicity and may indicate the development of HELLP syndrome. The onset of seizure activity indicates eclampsia. A serum magnesium level of 6.5 mEq/L would fall within the therapeutic range of 4 to 7 mEq/L.

diminished reflexes

A woman with severe preeclampsia is receiving magnesium sulfate. The woman serum magnesium level is 9.0mEq/L. Which finding would the nurse most likely note?

diminished reflexes Diminished or absent reflexes occur when a client develops magnesium toxicity, serum levels greater than 8.0 mEq/L. Elevated liver enzymes are unrelated to magnesium toxicity and may indicate the development of HELLP syndrome. The onset of seizure activity indicates eclampsia. A serum magnesium level of 6.5 mEq/L would fall within the therapeutic range of 4 to 7 mEq/L.

A woman with a recent incomplete abortion is to receive therapeutic misoprostol. The nurse understands that the rationale for administering this drug is to:

ensure passage of all the products of conception.

The nurse is required to assess a client for HELLP syndrome. Which are the signs and symptoms of this condition? Select all that apply.

epigastric pain upper right quadrant pain hyperbilirubinemia The signs and symptoms of HELLP syndrome are nausea, malaise, epigastric pain, upper right quadrant pain, demonstrable edema, and hyperbilirubinemia. Blood pressure higher than 160/110 mm Hg and oliguria are the symptoms of severe preeclampsia rather than HELLP syndrome.

The nurse is assessing a client at 12 weeks' gestation at a routine prenatal visit who reports something doesn't feel right. Which assessment findings should the nurse prioritize?

gestational hypertension, hyperemesis gravidarum, absence of FHR

A pregnant client has been admitted with reports of brownish vaginal bleeding. On examination there is an elevated hCG level, absent fetal heart sounds, and a discrepancy between the uterine size and the gestational age. The nurse interprets these findings to suggest which condition?

gestational trophoblastic disease

A nurse is providing care to a client who has been diagnosed with a common benign form of gestational trophoblastic disease. The nurse identifies this as:

hydatidiform mole.

The nurse should encourage a client with mild elevations in blood pressure to rest as much as possible in the lateral recumbent position to improve uteroplacental blood flow, reduce blood pressure, and promote diuresis. The nurse should maintain the client with severe preeclampsia on complete bed rest in the left lateral lying position. Keeping the head of the bed slightly elevated will not help to improve the condition of the client with mild elevations in blood pressure.

lateral recumbent position

The nurse is preparing the plan of care for a woman hospitalized for hyperemesis gravidarum. Which interventions would the nurse most likely include? Select all that apply.

maintaining NPO status for the first day or two administering antiemetic agents obtaining baseline blood electrolyte levels monitoring intake and output When hospitalization is necessary, oral food and fluids are withheld to allow the gut to rest. Antiemetic agents are ordered to help control nausea and vomiting. The woman is likely to be dehydrated, so the nurse would obtain baseline blood electrolyte levels and administer intravenous fluid and electrolyte replacement therapy as indicated. Once the nausea and vomiting subside, oral food and fluids are gradually reintroduced. Total parenteral nutrition or a feeding tube is used to prevent malnutrition only if the client does not improve with these interventions.

Which medication would the nurse prepare to administer if prescribed as treatment for an unruptured ectopic pregnancy?

methotrexate

A 28-year-old primigravida client with type 2 diabetes mellitus comes to the health care clinic for a routine first trimester visit reporting frequent episodes of fasting blood glucose levels being lower than normal, but glucose levels after meals being higher than normal. What should the nurse point out that these episodes are most likely related to?

normal response to the pregnancy This is a normal response to the pregnancy. During pregnancy, tissues become resistant to insulin to provide sufficient levels of glucose for the growing fetus. This can result in three normally occurring responses: blood glucose levels are lower than normal when fasting; blood glucose levels are higher than normal after meals; and insulin levels are increased after meals. The various hormones will prevent the mother from using most of the insulin produced to allow the extra glucose to get to the growing fetus.

A pregnant client with severe preeclampsia has developed HELLP syndrome. In addition to the observations necessary for preeclampsia, what other nursing intervention is critical for this client?

observation for bleeding Because of the low platelet count associated with this condition, women with HELLP syndrome need extremely close observation for bleeding, in addition to the observations necessary for preeclampsia. Maintaining a patent airway is a critical intervention needed for a client with eclampsia while she is having a seizure. Administration of a tocolytic would be appropriate for halting labor. Monitoring for infection is not a priority intervention in this situation.

The nurse is developing a plan of care for a woman who is pregnant with twins. The nurse includes interventions focusing on which area because of the woman's increased risk?

preeclampsia Women with multiple gestations are at high risk for preeclampsia, preterm labor, hydramnios, hyperemesis gravidarum, anemia, and antepartal hemorrhage. There is no association between multiple gestations and the development of chorioamnionitis.

A 44-year-old client has lost several pregnancies over the last 10 years. For the past 3 months, she has had fatigue, nausea, and vomiting. She visits the clinic and takes a pregnancy test; the results are positive. Physical examination confirms a uterus enlarged to 13 weeks' gestation; fetal heart tones are heard. Ultrasound reveals that the client is experiencing some bleeding. Considering the client's prenatal history and age, what does the nurse recognize as the greatest risk for the client at this time?

pregnancy loss The client's advanced maternal age (pregnancy in a woman 35 years or older) increases her risk for pregnancy loss. Hypertension, preterm labor, and prematurity are risks as this pregnancy continues. Her greatest risk at 13 weeks' gestation is losing this pregnancy.

A nurse is explaining to a group of nursing students that eclampsia or seizures in pregnant women are preceded by an acute increase in maternal blood pressure. What are features of an acute increase in blood pressure? Select all that apply.

proteinuria hypereflexia blurring of vision Eclampsia is usually preceded by an acute increase in blood pressure as well as worsening signs of multi-organ system failure seen as increasing liver enzymes, proteinuria, and symptoms such as blurred vision and hyperreflexia. Hyperglycemia and auditory hallucinations are not seen with an acute increase in maternal blood pressure.

A nurse is assessing a client diagnosed with mild preeclampsia. The nurse suspects that the client has developed severe preeclampsia based on which finding?

urine output of less than 400 mL/24 hours Severe preeclampsia may develop suddenly and bring with it high blood pressure of more than 160/110 mm Hg, proteinuria of more than 5 g in 24 hours, oliguria of less than 400 mL in 24 hours, cerebral and visual symptoms, and rapid weight gain. Mild facial edema or hand edema occurs with mild preeclampsia. Proteinuria in severe preeclampsia is greater than 500 mg/24 hours.

A pregnant woman is admitted to the hospital with a diagnosis of placenta previa. Which action would be the priority for this woman on admission?

Assessing fetal heart tones by use of an external monitor

A nurse is preparing a nursing care plan for a client who is admitted at 22 weeks' gestation with advanced cervical dilatation to 5 cm, cervical insufficiency, and a visible amniotic sac at the cervical opening. Which primary goal should the nurse prioritize at this point?

Bed rest to maintain pregnancy as long as possible

A client has been admitted to the hospital with a diagnosis of severe preeclampsia. Which nursing intervention is the priority?

Confine the client to bed rest in a darkened room. With severe preeclampsia, most women are hospitalized so that bed rest can be enforced and a woman can be observed more closely than she can be on home care. The nurse should darken the room if possible because a bright light can also trigger seizures. The other interventions listed pertain to a client who has experienced a seizure and has thus progressed to eclampsia.

What special interventions would the nurse implement in a client who is carrying twin fetuses?

Demonstrate to the client how to perform fetal movement counts after 32 weeks.

A woman with a recent incomplete abortion is to receive therapeutic misoprostol. The nurse understands that the rationale for administering this drug is to:

Ensure passage of all the products of conception

Spontaneous miscarriage occurs in 5% to 10% of all pregnancies.

False Spontaneous miscarriage occurs in 15% to 20% of all pregnancies and arises from natural causes.

Current research indicates that supplementation with what before pregnancy may reduce the risk of placental abruption?

Folic Acid

The nurse is caring for a woman at 32 weeks gestation with severe preeclampsia. Which assessment finding should the nurse prioritize after the administration of hydralazine to this client?

Tachycardia

A pregnant patient with a history of premature cervical dilatation undergoes cervical cerclage. Which outcome indicates that this procedure has been successful?

The client delivers a full-term fetus at 39 weeks' gestation.

A young mother gives birth to twin boys who shared the same placenta. What serious complication are they at risk for? Twin-to-twin transfusion syndrome (TTTS) ABO incompatibility TORCH syndrome HELLP syndrome

Twin-to-twin transfusion syndrome (TTTS) Explanation: When twins share a placenta, a serious condition called twin-to-twin transfusion syndrome (TTTS) can occur.

A pregnant woman with preeclampsia is to receive magnesium sulfate IV. Which assessment would be most important prior to administering a new dose?

patellar reflex A symptom of magnesium sulfate toxicity is loss of deep tendon reflexes. Assessing for one of these before administration is assurance the drug administration will be safe.

A nurse is caring for a client undergoing treatment for ectopic pregnancy. Which symptom is observed in a client if rupture or hemorrhaging occurs before the ectopic pregnancy is successfully treated?

phrenic nerve irritation The symptoms if rupture or hemorrhaging occurs before successfully treating the pregnancy are lower abdomen pain, feelings of faintness, phrenic nerve irritation, hypotension, marked abdominal tenderness with distension, and hypovolemic shock. Painless bright red vaginal bleeding occurring during the second or third trimester is the clinical manifestation of placenta previa. Fetal distress and tetanic contractions are not the symptoms observed in a client if rupture or hemorrhaging occurs before successfully treating an ectopic pregnancy.

A woman in labor has sharp fundal pain accompanied by slight vaginal bleeding. What is the most likely cause of these symptoms?

premature separation of the placenta Premature separation of the placenta begins with sharp fundal pain, usually followed by vaginal bleeding. Placenta previa usually produces painless bleeding. Preterm labor contractions are more often described as cramping. Possible fetal death or injury does not present with sharp fundal pain. It is usually painless.

A nurse in the maternity triage unit is caring for a client with a suspected ectopic pregnancy. Which nursing intervention should the nurse perform first?

Assess the client's vital signs.

A pregnant patient is diagnosed with preterm labor. What should the nurse teach the patient to help prevent the reoccurrence of preterm labor? Select all that apply.

Drink 8 to 10 glasses of fluid each day. Report any signs of ruptured membranes. Remain on bed rest except to use the bathroom. To reduce the onset of preterm labor, the nurse should instruct the patient to drink 8 to 10 glasses of fluid each day to remain hydrated. The patient should also report any signs of ruptured membranes and remain on bed rest unless using the bathroom. Should uterine contractions begin, the patient should be instructed to lie on either the right or left side to increase blood return to the uterus. The patient should not engage in any activity other than bed rest with bathroom privileges.

A nurse is working as part of a community group focusing efforts on preventing isoimmunization during pregnancy at the local women's health clinic. Which measure would the group encourage?

Rho(D) immune globulin administration to Rh-negative women Rh incompatibility can be prevented with the use of Rho(D) immune globulin. Hemolysis associated with ABO incompatibility is limited to mothers with type O blood and their fetuses with type A or B blood. Amniocentesis would be appropriate for treatment of polyhydramnios not isoimmunization. Cerclage is a treatment for cervical insufficiency.

The nurse is caring for a client who has a multifetal pregnancy. What topic should the nurse prioritize during health education? Risk for blood incompatibilities Risk for hypertension Parenting skills Signs of preterm labor

Signs of preterm labor Explanation: The client with a multifetal pregnancy must be made aware of the risks posed by preterm labor. There is no corresponding increase in the risk for hypertension or blood incompatabilities. Parenting skills are secondary to physiologic needs at this point.

The nurse is monitoring a pregnant patient who is receiving intravenous magnesium sulfate for eclampsia. During the last assessment, the nurse was unable to elicit a patellar reflex. What should the nurse do?

Stop the current infusion. When infusing magnesium sulfate, the nurse should stop the infusion if deep tendon reflexes are absent. Checking the fetal heart rate and measuring blood pressure could waste time and provide the patient with more magnesium sulfate. The infusion rate should not be increased because this could lead to cardiac dysrhythmias and respiratory depression.

Which medication would the nurse prepare to administer if prescribed as treatment for an unruptured ectopic pregnancy?

methotrexate Methotrexate, a folic acid antagonist that inhibits cell division in the developing embryo, is most commonly used to treat ectopic pregnancy. Oxytocin is used to stimulate uterine contractions and would be inappropriate for use with an ectopic pregnancy. Promethazine and ondansetron are antiemetics that may be used to treat hyperemesis gravidarum.

The nurse is is giving discharge instructions to a client who experienced a complete spontaneous abortion. Which question should the nurse prioritize at this time?

"Do you have someone to talk to, or may I give you the names and numbers for some possible grief counselors?" When a woman has a spontaneous abortion, or miscarriage, one important consideration is the emotional needs of the woman once she is home. She may not want to talk about the loss for a period of time, but the nurse needs to determine her support system for the future. Asking the woman if she is "going to try again" is an inappropriate question for the nurse to ask and diminishes the experience of having a spontaneous abortion. It would be inappropriate to point out the woman is not the only one to have this experience or to offer ways to prevent it from happening in the future. The woman needs to deal with this situation first before moving on to a possible "next" time.

A nurse is caring for a young woman who is in her 10th week of gestation. She comes into the clinic reporting vaginal bleeding. Which assessment finding best correlates with a diagnosis of hydatidiform mole?

Dark red, "clumpy" vaginal discharge If a complete molar pregnancy continues into the second trimester undetected, other signs and symptoms appear. The woman often presents with complaints of dark to bright red vaginal bleeding and pelvic pain. Infrequently, she will report passage of grapelike vesicles.

A 28-year-old client with a history of endometriosis presents to the emergency department with severe abdominal pain and nausea and vomiting. The client also reports her periods are irregular with the last one being 2 months ago. The nurse prepares to assess for which possible cause for this client's complaints?

Ectopic pregnancy The most commonly reported symptoms of ectopic pregnancy are pelvic pain and/or vaginal spotting. Other symptoms of early pregnancy, such as breast tenderness, nausea, and vomiting, may also be present. The diagnosis is not always immediately apparent because many women present with complaints of diffuse abdominal pain and minimal to no vaginal bleeding. Steps are taken to diagnose the disorder and rule out other causes of abdominal pain. Given the history of the client and the amount of pain, the possibility of ectopic pregnancy needs to be considered. A healthy pregnancy would not present with severe abdominal pain unless the client were term and she was in labor. With a molar pregnancy the woman typically presents between 8 to 16 weeks' gestation reporting painless (usually) brown to bright red vaginal bleeding. Placenta previa typically presents with painless, bright red bleeding that begins with no warning.

A woman with severe preeclampsia is receiving magnesium sulfate. The woman's serum magnesium level is 9.0 mEq/L. Which finding would the nurse most likely note?

diminished reflexes Diminished or absent reflexes occur when a client develops magnesium toxicity, serum levels greater than 8.0 mEq/L. Elevated liver enzymes are unrelated to magnesium toxicity and may indicate the development of HELLP syndrome. The onset of seizure activity indicates eclampsia. A serum magnesium level of 6.5 mEq/L would fall within the therapeutic range of 4 to 7 mEq/L.

During a routine prenatal visit, a client is found to have proteinuria and a blood pressure rise to 140/90 mm Hg. The nurse recognizes that the client has which condition?

mild preeclampsia A woman is said to have gestational hypertension when she develops an elevated blood pressure (140/90 mm Hg) but has no proteinuria or edema. If a seizure from gestational hypertension occurs, a woman has eclampsia, but any status above gestational hypertension and below a point of seizures is preeclampsia. A woman is said to be mildly preeclamptic when she has proteinuria and a blood pressure rise to 140/90 mm Hg, taken on two occasions at least 6 hours apart. A woman has passed from mild to severe preeclampsia when her blood pressure rises to 160 mm Hg systolic and 110 mm Hg diastolic or above on at least two occasions 6 hours apart at bed rest (the position in which blood pressure is lowest) or her diastolic pressure is 30 mm Hg above her prepregnancy level. Marked proteinuria, 3+ or 4+ on a random urine sample or more than 5 g in a 24-hour sample, and extensive edema are also present. A woman has passed into eclampsia when cerebral edema is so acute a grand-mal seizure (tonic-clonic) or coma has occurred.

A client with preeclampsia is receiving magnesium sulfate. Which nursing assessment should be ongoing while the medication is being administered?

respiratory rate The level of magnesium in therapeutic range is 4 to 8 mg/dL. If magnesium toxicity occurs, one sign in the client will be a decrease in the respiratory rate and a potential respiratory arrest. Respiratory rate will be monitored when on this medication. The client's hemoglobin and ability to sleep are not factors for ongoing assessments for the client on magnesium sulfate. Urinary output is measured hourly on the preeclamptic client receiving magnesium sulfate, but urine protein is not an ongoing assessment.

A woman at 8 weeks' gestation is admitted for ectopic pregnancy. She is asking why this has occurred. The nurse knows that which factor is a known risk factor for ectopic pregnancy?

use of IUD for contraception

A woman at 8 weeks' gestation is admitted for ectopic pregnancy. She is asking why this has occurred. The nurse knows that which factor is a known risk factor for ectopic pregnancy?

use of IUD for contraception Use of an IUD with progesterone has a known increased risk for development of ectopic pregnancies. The nurse needs to complete a full history of the client to determine if she had any other risk factors for an ectopic pregnancy. Adhesions, scarring, and narrowing of the tubal lumen may block the zygote's progress to the uterus. Any condition or surgical procedure that can injure a fallopian tube increases the risk. Examples include salpingitis, infection of the fallopian tube, endometriosis, history of prior ectopic pregnancy, any type of tubal surgery, congenital malformation of the tube, and multiple elective abortions. Conditions that inhibit peristalsis of the tube can result in tubal pregnancy. Hormonal factors may play a role because tubal pregnancy occurs more frequently in women who take fertility drugs or who use progesterone intrauterine contraceptive devices (IUDs). A high number of pregnancies, multiple gestation pregnancy, and the use of oral contraceptives are not known risk factors for ectopic pregnancy.

A 35-year-old client is seen for her 2-week postoperative appointment after a suction curettage was performed to evacuate a hydatidiform mole. The nurse explains that the human chorionic gonadotropin (hCG) levels will be reviewed every 2 weeks and teaches about the need for reliable contraception for the next 6 months to a year. The client states, "I'm 35 already. Why do I have to wait that long to get pregnant again?" What is the nurse's best response?

"A contraceptive is used so that a positive pregnancy test resulting from a new pregnancy will not be confused with the increased level of hCG that occurs with a developing malignancy." Because of the risk of choriocarcinoma, the woman receives extensive treatment. Therapy includes baseline chest X-ray to detect lung metastasis physical exam including pelvic exam. Serum B-hCG levels weekly until negative results are obtained three consecutive times, then monthly for 6 to 12 months. The woman is cautioned to avoid pregnancy during this time because the increasing B-hCG levels associated with pregnancy would cause confusion as to whether cancer had developed. If after a year B-hCG seruim titers are within normal levels, a normal pregnancy can be achieved.

A woman of 16 weeks' gestation telephones the nurse because she has passed some "berry-like" blood clots and now has continued dark brown vaginal bleeding. Which action would the nurse instruct the woman to do?

"Come to the health facility with any vaginal material passed."

A woman at 31 weeks' gestation presents to the emergency department with bright red vaginal bleeding, reporting that the onset of the bleeding was sudden and without pain. Which diagnostic test should the nurse prioritize?

A transvaginal ultrasound

A client at 11 weeks' gestation experiences pregnancy loss. The client asks the nurse if the bleeding and cramping that occurred during the miscarriage were caused by working long hours in a stressful environment. What is the most appropriate response from the nurse?

"I can understand your need to find an answer to what caused this. Let's talk about this further."

A client at 11 weeks' gestation experiences pregnancy loss. The client asks the nurse if the bleeding and cramping that occurred during the miscarriage were caused by working long hours in a stressful environment. What is the most appropriate response from the nurse? "Your spontaneous bleeding is not work-related." "It is hard to know why a woman bleeds during early pregnancy." "Something was wrong with the fetus." "I can understand your need to find an answer to what caused this. Let's talk about this further."

"I can understand your need to find an answer to what caused this. Let's talk about this further." Explanation: Talking with the client may assist her to explore her feelings. She and her family may search for a cause for a spontaneous early bleeding so they can plan for future pregnancies. Even with modern technology and medical advances, however, a direct cause cannot usually be determined.

A client at 11 weeks' gestation experiences pregnancy loss. The client asks the nurse if the bleeding and cramping that occurred during the miscarriage were caused by working long hours in a stressful environment. What is the most appropriate response from the nurse?

"I can understand your need to find an answer to what caused this. Let's talk about this further." Talking with the client may assist her to explore her feelings. She and her family may search for a cause for a spontaneous early bleeding so they can plan for future pregnancies. Even with modern technology and medical advances, however, a direct cause cannot usually be determined.

Which statement by a pregnant client indicates the need for more teaching about preeclampsia?

"If I have changes in my vision, I will lie down and rest." Changes in the visual field may indicate the client has moved from preeclampsia to severe preeclampisa and is at risk for developing a seizure due to changes in cerebral blood flow. The client would require immediate assessment and intervention. The other options would not indicate that more teaching about preeclampsia is indicated.

A client with a history of cervical insufficiency is seen for reports of pink-tinged discharge and pelvic pressure. The primary care provider decides to perform a cervical cerclage. The nurse teaches the client about the procedure. Which client response indicates that the teaching has been effective?

"Purse-string sutures are placed in the cervix to prevent it from dilating." The cerclage, or purse string suture is inserted into the cervix to prevent preterm cervical dilatation and pregnancy loss. Staples, glue, or a cervical cap will not prevent the cervix from dilating.

A client in her first trimester arrives at the emergency room with reports of severe cramping and vaginal spotting. On examination, the health care provider informs her that no fetal heart sounds are evident and orders a dilatation and curettage. The client looks frightened and confused and states that she does not believe in abortion. Which statement by the nurse is best?

"Unfortunately, the pregnancy is already lost. The procedure is to clear the uterus to prevent further complications."

A nurse has been assigned to assess a pregnant client for abruptio placenta. For which classic manifestation of this condition should the nurse assess?

"knife-like" abdominal pain with vaginal bleeding The classic manifestations of abruption placenta are painful dark red vaginal bleeding, "knife-like" abdominal pain, uterine tenderness, contractions, and decreased fetal movement. Painless bright red vaginal bleeding is the clinical manifestation of placenta previa. Generalized vasospasm is the clinical manifestation of preeclampsia and not of abruptio placenta.

A client in her 20th week of gestation develops HELLP syndrome. What are features of HELLP syndrome? Select all that apply.

1. hemolysis 2. elevated liver enzymes 3. low platelet count

The client arrives in the office and reports that her feet and legs swelling. During a client evaluation, the nurse notes that she can elicit a 4-mm skin depression that disappears in 10 to 15 seconds. The nurse correctly documents this finding as:

2+ pitting edema. Pitting edema is recorded using the following relative scale: 1+ is a 2-mm depression that disappears rapidly; 2+ is a 4-mm depression that disappears in 10 to 15 seconds; 3+ is a 6-mm depression that lasts more than one minute; and 4+ is an 8-mm depression that lasts 2 to 3 minutes.

During pregnancy a woman's blood volume increases to accommodate the growing fetus to the point that vital signs may remain within normal range without showing signs of shock until the woman has lost what percentage of her blood volume?

40% Vital signs can be within normal range, even with significant blood loss, because a pregnant woman can lose up to 40% of her total blood volume without showing signs of shock.

A nurse is caring for a client with hyperemesis gravidarum. Which nursing action is the priority for this client?

Administer IV NS with vitamins and electrolytes

When caring for a client with premature rupture of membranes (PROM), the nurse observes an increase in the client's pulse. What should the nurse do next?

Assess the client's temperature. A temperature elevation or an increase in the pulse of a client with PROM would indicate infection. Increase in the pulse does not indicate preterm labor or cord compression. The nurse should monitor FHR patterns continuously, reporting any variable decelerations suggesting cord compression. Respiratory distress syndrome is one of the perinatal risks associated with PROM.

A nurse in the maternity triage unit is caring for a client with a suspected ectopic pregnancy. Which nursing intervention should the nurse perform first?

Assess the client's vital signs. A suspected ectopic pregnancy can put the client at risk for hypovolemic shock. The assessment of vital signs should be performed first, followed by any procedures to maintain the ABCs. Providing emotional support would also occur, as would obtaining a surgical consent, if needed, but these are not first steps.

A patient in labor and delivery has just been diagnosed with pre-eclampsia. Which signs and symptom should the nurse prioritize when assessing this client? Select all that apply.

BP 140/90 mm Hg edema of face headache Clinical manifestations of preeclampsia include hypertension of greater than 140 mm Hg systolic or 90 mm Hg diastolic, protein in the urine, edema, severe headache, hyperactive deep tendon reflexes (not slow) and clonus, blurred or double vision, and nausea and pain int he epigastric region. Glucose in the urine would possibly indicate diabetes. TAKE A PRACTICE QUIZ

The nurse is comforting and listening to a young couple who just suffered a miscarriage. When asked why this happened, which reason should the nurse share as a common cause?

Chromosomal abnormality

The nurse is orientating in the Labor and Delivery unit and asks her preceptor how to differentiate a client with preeclampsia from one with eclampsia. Which symptoms would the preceptor describe to the new nurse as indicative of severe preeclampsia? Select all that apply.

Blood pressure above 160/110 mm Hg Nondependent edema Hyperactive deep tendon reflexes Preeclampsia occurs when a pregnant woman develops hypertension occurring after 20 weeks gestation and only resolves after the fetus is delivered. Preeclampsia is exhibited by 2+ or more proteinuria, nondependent edema, blood pressure greater than 140 mm Hg systolic and above 90 mm Hg diastolic, and CNS irritability demonstrated by hyperactive deep tendon reflexes. If the client has a seizure, he has moved to eclampsia. Glycosuria is not associated with preeclampsia.

The nurse is comforting and listening to a young couple who just suffered a miscarriage. When asked why this happened, which reason should the nurse share as a common cause?

Chromosomal abnormality The most common cause for the loss of a fetus in the first trimester is associated with a genetic defect or chromosomal abnormality. There is nothing that can be done, and the mother should feel no fault. The nurse needs to encourage the parents to speak with a health care provider for further information and questions related to genetic testing. Early pregnancy loss is not associated with maternal smoking, lack of prenatal care, or the age of the mother.

Which nursing diagnosis would be most appropriate for a woman diagnosed with gestational hypertension?

Deficient fluid volume related to vasospasm of arteries Gestational hypertension is caused by vascular spasm. This leads to increased blood pressure and edema. Extensive edema leads to a deficiency of fluid volume.

What special interventions would the nurse implement in a client who is carrying twin fetuses?

Demonstrate to the client how to perform fetal movement counts after 32 weeks. A woman carrying a multiple gestation needs to keep up with how her fetuses are doing, and an excellent way to do that is by doing fetal movement counts, or "kick counts" as they are sometimes called. This starts at around 32 weeks' gestation for an uncomplicated pregnancy and continues until delivery. Weekly or bi-weekly NSTs begin after 32 weeks. Obstetrical ultrasounds are done every 4 to 6 weeks after confirmation of a multiple fetal pregnancy. The client needs to increase her intake, along with her iron and folic acid intake, to provide adequate nutrition for both fetuses.

A woman at 37 weeks gestation presents to the labor and delivery area with symptoms of abruptio placentae. Which action should the nurse prioritize?

Ensure large bore IV access is obtained

Current research indicates that supplementation with what before pregnancy may reduce the risk of placental abruption?

Folic acid

A patient is admitted with a diagnosis of ectopic pregnancy. For what should the nurse anticipate preparing the patient?

Immediate surgery An ectopic pregnancy is one in which implantation occurred outside the uterine cavity, usually within the fallopian tube. As the embryo grows, the fallopian tube can rupture. The therapy for ruptured ectopic pregnancy is laparoscopy to ligate the bleeding vessels and to remove or repair the damaged fallopian tube. There is no reason to begin uterine monitoring. The patient does not need to be on bed rest for 4 weeks. A tocolytic is not needed because the patient is not in labor.

A pregnant patient is being admitted for severe preeclampsia. In which room location should the nurse place this patient?

In the back private room With severe preeclampsia, hospitalization is required so that bed rest can be enforced and the patient can be observed more closely. A patient with severe preeclampsia is admitted to a private room so that rest is undisturbed. Noises such as a baby crying, elevator doors opening and closing, and conversation from the nurse's station is sufficient to trigger a seizure. A private room will help reduce the likelihood of seizure development.

A patient recovering from an uneventful vaginal delivery is prescribed Rho(D) immune globulin (RhIG). What should the nurse explain to the patient regarding the purpose of this medication?

It prevents maternal D antibody formation.

The nurse is caring for a pregnant client with severe preeclampsia. Which nursing intervention should a nurse perform to institute and maintain seizure precautions in this client?

Keep the suction equipment readily available.

A 25-week-gestation client presents with a blood pressure of 142/90, pulse 78, no edema, and urine negative for protein. What would the nurse do next?

Notify the healthcare provider The client is exhibiting a sign of gestational hypertension, elevated blood pressure greater than or equal to 140/90 mm Hg that develops for the first time during pregnancy. The healthcare provider should be notified to assess the client. Without the presence of edema or protein in the urine, the client does not have preeclampsia.

A 32-year-old gravida 3 para 2 at 36 weeks' gestation comes to the obstetric department reporting abdominal pain. Her blood pressure is 164/90 mm/Hg, her pulse is 100 beats per minute, and her respirations are 24 per minute. She is restless and slightly diaphoretic with a small amount of dark red vaginal bleeding. What assessment should the nurse make next?

Palpate the fundus and check fetal heart rate.

A woman in labor suddenly reports sharp fundal pain accompanied by slight dark red vaginal bleeding. The nurse should prepare to assist with which situation?

Premature separation of the placenta

A woman in labor suddenly reports sharp fundal pain accompanied by slight dark red vaginal bleeding. The nurse should prepare to assist with which situation?

Premature separation of the placenta Premature separation of the placenta begins with sharp fundal pain, usually followed by dark red vaginal bleeding. Placenta previa usually produces painless bright red bleeding. Preterm labor contractions are more often described as cramping. Possible fetal death or injury does not present with sharp fundal pain. It is usually painless.

A client at 37 weeks' gestation presents to the emergency department with a BP 150/108 mm Hg, 1+ pedal edema, 1+ proteinuria, and normal deep tendon reflexes. Which assessment should the nurse prioritize as the client is administered magnesium sulfate IV?

Respiratory rate The level of magnesium in therapeutic range is 4 to 8 mg/dL. If magnesium toxicity occurs, one sign in the client will be a decrease in the respiratory rate and a potential respiratory arrest. Respiratory rate will be monitored when on this medication. The client's hemoglobin and ability to sleep are not factors for ongoing assessments for the client on magnesium sulfate. Urinary output is measured hourly on the preeclamptic client receiving magnesium sulfate, but urine protein is not an ongoing assessment.

A nurse is conducting a presentation for a group of pregnant women about conditions that can occur during pregnancy and that place the woman at high-risk. When discussing blood incompatibilities, which measure would the nurse explain as most effective in preventing isoimmunization during pregnancy?

Rho(D) immune globulin administration to Rh-negative women

A nurse is conducting a presentation for a group of pregnant women about conditions that can occur during pregnancy and that place the woman at high-risk. When discussing blood incompatibilities, which measure would the nurse explain as most effective in preventing isoimmunization during pregnancy?

Rho(D) immune globulin administration to Rh-negative women Rh incompatibility can be prevented with the use of Rho(D) immune globulin. Hemolysis associated with ABO incompatibility is limited to mothers with type O blood and their fetuses with type A or B blood. Amniocentesis would be appropriate for treatment of polyhydramnios, not isoimmunization. Cerclage is a treatment for cervical insufficiency.

A nurse is describing the use of Rho(D) immune globulin as the therapy of choice for isoimmunization in Rh-negative women and for other conditions to a group of nurses working at the women's health clinic. The nurse determines that additional teaching is needed when the group identifies which situation as an indication for Rho(D) immune globulin?

STIs Indications for Rho(D) immune globulin include isoimmunization, ectopic pregnancy, chorionic villus sampling, amniocentesis, prenatal hemorrhage, molar pregnancy, maternal trauma, percutaneous umbilical sampling, therapeutic or spontaneous abortion, fetal death, or fetal surgery.

A 25-year-old pregnant client has just been diagnosed with hyperemesis gravidarum. Which instruction should the nurse prioritize during a teaching session?

Take your anti-nausea medicine around the clock. Antiemetics, if prescribed, are usually more effective when given on a regular, around-the-clock schedule versus as-needed (PRN) dosing. Increasing fluid intake may exacerbate nausea. It would be better if the woman did not drink a lot of fluids when she ate. She should continue with a balanced, nutritious diet, consulting a dietitian if needed. Doing her own cooking will not assist with developing a tolerance to the odors but will help her eliminate odors which might trigger her.

In returning to the hospital floor after a weekend off, the nurse takes over care of a pregnant client who is resting in a darkened room. The client is receiving betamethasone and magnesium sulfate. What could the nurse deduce from those findings?

The client is suffering from severe preeclampsia, and the care team is attempting to prevent advancement of the disorder to eclampsia; they are attempting to help the baby's lungs mature quickly so that they can have the baby be born as soon as possible. The administration of magnesium sulfate is to relax the skeletal muscles and raise the threshold for a seizure. The administration of the betamethasone is to try and hasten the maturity of the fetus' lungs for birth. This woman is in advanced preeclampsia and must be monitored for progression to eclampsia. The scenario described does not indicate a client with hypertension who may be discharged home once the condition is under control. A woman in eclampsia would either be seizing or comatose, not resting in a quiet room. Symptoms of mild preeclampsia are limited to slightly elevated blood pressure and small amounts of protein in the urine. Betamethasone may be indicated at this time.

A young mother gives birth to twin boys who shared the same placenta. What serious complication are they at risk for?

Twin-to-twin transfusion syndrome (TTTS) When twins share a placenta, a serious condition called twin-to-twin transfusion syndrome (TTTS) can occur.

A nurse is assessing pregnant clients for the risk of placenta previa. Which client faces the greatest risk for this condition?

a client who had a myomectomy to remove fibroids

The nurse must stress to which client being discharged for home the critical need to return for monthly follow-up visits?

a woman who has experienced a molar pregnancy. Molar pregnancies can indicate the possibility of developing malignancy. The woman will need close observation and follow-up for the year following the diagnosis. Follow-up visits after an ectopic pregnancy or a complete spontaneous abortion are typically scheduled at six weeks, not monthly. A woman who is Rh negative does not need a follow-up visit because of her Rh status.

A nurse is caring for a client who just experienced a miscarriage in her first trimester. When asked by the client why this happened, which is the best response from the nurse?

abnormal fetal development The most frequent cause of miscarriage in the first trimester of pregnancy is abnormal fetal development, due either to a teratogenic factor or to a chromosomal aberration. In other miscarriages, immunologic factors may be present or rejection of the embryo through an immune response may occur. Another common cause of early miscarriage involves implantation abnormalities. Miscarriage may also occur if the corpus luteum on the ovary fails to produce enough progesterone to maintain the decidua basalis.

Rho(D) immune globulin will be prescribed for an Rh negative mother undergoing which test?

amniocentesis Amniocentesis is a procedure requiring a needle to enter into the amniotic sac. There is a risk of mixing of the fetal and maternal blood which could result in blood incompatibility. A contraction test, a nonstress test, and biophysical profile are not invasive, so there would be no indication for Rho(D) immune globulin to be administered.

It is determined that a client's blood Rh is negative and her partner's is positive. To help prevent Rh isoimmunization, the nurse would expect to administer Rho(D) immune globulin at which time? 24 hours before birth and 24 hours after birth at 28 weeks' gestation and again within 72 hours after birth at 32 weeks' gestation and immediately before discharge in the first trimester and within 2 hours of birth

at 28 weeks' gestation and again within 72 hours after birth Explanation: To prevent isoimmunization, the woman should receive Rho(D) immune globulin at 28 weeks and again within 72 hours after birth.

A client is admitted at 22 weeks' gestation with advanced cervical dilatation to 5 centimeters, cervical insufficiency, and a visible amniotic sac at the cervical opening. What is the primary goal for this client at this point?

bed rest to maintain pregnancy as long as possible At 22 weeks' gestation, the fetus is not viable. The woman would be placed on bed rest, total, with every attempt made to halt any further progression of dilatation as long as possible. The nurse would not want this fetus to be born vaginally at this stage of gestation. It is not the nurse's responsibility to notify the client's social support of a possible loss of the pregnancy. It is not appropriate at this time to educate the mother on causes of cervical insufficiency for future pregnancies.

A pregnant client at 20 weeks' gestation arrives at the health care facility reporting excessive vaginal bleeding and no fetal movements. Which assessment finding would the nurse anticipate in this situation?

cervical incompetence This client has reported symptoms of a spontaneous abortion or miscarriage of the second trimester. Miscarriages in the second trimester are most often related to maternal factors such as cervical incompetence, congenital or acquired anomaly of the uterine cavity, hypothyroidism, diabetes mellitus, chronic nephritis, use of crack cocaine, inherited and acquired thrombophilias, lupus, polycystic ovary syndrome, severe hypertension, and acute infection such as rubella virus, cytomegalovirus, herpes simplex virus, bacterial vaginosis, and toxoplasmosis. Cervical incompetence is a condition where there is painless cervical dilatation and results in second trimester fetal loss or can progress to preterm premature rupture of membranes. Ectopic pregnancy, congenital malformations, and placenta previa are not involved in causing second trimester fetal loss. Ectopic pregnancy usually leads to first trimester fetal loss. Placenta previa is a condition in which there is implantation of the placenta to the lower uterine segment. Congenital malformations result in first trimester fetal loss.

A woman with an incomplete abortion is to receive misoprostol. The woman asks the nurse, "Why am I getting this drug?" The nurse responds to the client, integrating understanding that this drug achieves which effect?

ensures passage of all the products of conception

A nurse is assessing a pregnant client for the possibility of preexisting conditions that could lead to complications during pregnancy. The nurse suspects that the woman is at risk for hydramnios based on which preexisting condition?

diabetes Approximately 18% of all women with diabetes will develop hydramnios during their pregnancy. Hydramnios occurs in approximately 3% of all pregnancies and is associated with fetal anomalies of development.

A woman with severe preeclampsia is receiving magnesium sulfate. The woman's serum magnesium level is 9.0 mEq/L. Which finding would the nurse most likely note?

diminished reflexes

A pregnant woman is being evaluated for HELLP. The nurse reviews the client's diagnostic test results. Which result would the nurse interpret as helping to confirm this diagnosis?

elevated LDH HELLP (hemolysis, elevated liver enzymes, low platelet count) syndrome is a variant of the preeclampsia/eclampsia syndrome. The diagnosis is based on laboratory test results, including: low hematocrit, elevated LDH, elevated AST, elevated ALT, elevated BUN, elevated bilirubin level, elevated uric acid and creatinine levels, and low platelet count. White blood cell counts are not used to evaluate for HELLP.

A woman with an incomplete abortion is to receive misoprostol. The woman asks the nurse, "Why am I getting this drug?" The nurse responds to the client, integrating understanding that this drug achieves which effect? ensures passage of all the products of conception alleviates strong uterine cramping halts the progression of the abortion suppresses the immune response to prevent isoimmunization

ensures passage of all the products of conception Explanation: Misoprostol is used to stimulate uterine contractions and evacuate the uterus after an abortion to ensure passage of all the products of conception. Rho(D) immune globulin is used to suppress the immune response and prevent isoimmunization.

A client has come to the office for a prenatal visit during her 22nd week of gestation. On examination, it is noted that her blood pressure has increased to 138/90 mm Hg. Her urine is negative for proteinuria. The nurse recognizes which factor as the potential cause?

gestational hypertension Gestational hypertension is characterized by hypertension without proteinuria after 20 weeks of gestation resolving by 12 weeks postpartum. It is defined as systolic blood pressure of greater than 140 mm Hg and/or diastolic of greater than 90 mm Hg on at least two occasions at least 6 hours apart after the 20th week of gestation, in women known to be normotensive prior to this time and prior to pregnancy. HELLP is an acronym that refers to hemolysis, elevated liver enzymes, and low platelets. Preeclampsia may result if hypertension is not controlled or advances to a more severe state.

A nurse is providing care to a client who has been diagnosed with a common benign form of gestational trophoblastic disease. The nurse identifies this as:

hydatidiform mole. Gestational trophoblastic disease comprises a spectrum of neoplastic disorders that originate in the placenta. The two most common types are hydatidiform mole (partial or complete) and choriocarcinoma. Ectopic pregnancy, placenta accreta, and hydramnios fall into different categories of potential pregnancy complications.

A woman is admitted with a diagnosis of ectopic pregnancy. For which procedure should the nurse prepare?

immediate surgery

Some women experience a rupture of their membranes before going into true labor. A nurse recognizes that a woman who presents with PPROM has completed how many weeks of gestation?

less than 37 weeks Preterm premature rupture of membranes (PPROM) is defined as the rupture of the membranes prior to the onset of labor in a woman who is less than 37 weeks' gestation. PROM (premature rupture of membranes) refers to a woman who is beyond 37 weeks' gestation, has presented with spontaneous rupture of the membranes, and is not in labor.

A nurse is reviewing the medical record of a pregnant client diagnosed with placenta previa. The physical exam reveals that the placenta is implanted near the internal os but does not reach it. The nurse interprets this as which type of placenta previa?

low-lying Placenta previa is generally classified according to the degree of coverage or proximity to the internal os, as follows: total placenta previa - the internal cervical os is completely covered by the placenta; partial placenta previa - the internal os is partially covered by the placenta; marginal placenta previa - the placenta is at the margin or edge of the internal os; low-lying placenta previa - the placenta is implanted in the lower uterine segment and is near the internal os but does not reach it.

Which medication would the nurse prepare to administer if prescribed as treatment for an unruptured ectopic pregnancy? promethazine oxytocin methotrexate ondansetron

methotrexate Explanation: Methotrexate, a folic acid antagonist that inhibits cell division in the developing embryo, is most commonly used to treat ectopic pregnancy. Oxytocin is used to stimulate uterine contractions and would be inappropriate for use with an ectopic pregnancy. Promethazine and ondansetron are antiemetics that may be used to treat hyperemesis gravidarum.

After an examination, a client has been determined to have an unruptured ectopic pregnancy. Which medication would the nurse anticipate being prescribed?

methotrexate Methotrexate, a folic acid antagonist that inhibits cell division in the developing embryo, is most commonly used to treat ectopic pregnancy. Oxytocin is used to stimulate uterine contractions and would be inappropriate for use with an ectopic pregnancy. Promethazine and ondansetron are antiemetics that may be used to treat hyperemesis gravidarum.

A woman in week 35 of her pregnancy with severe hydramnios is admitted to the hospital. The nurse recognizes that which concern is greatest regarding this client?

preterm rupture of membranes followed by preterm birth

A client is admitted to labor and birth for management of severe preeclampsia. An IV infusion of magnesium sulfate is started. What is the primary goal for magnesium sulfate therapy?

prevent maternal seizures The primary therapy goal for any preeclamptic client is to prevent maternal seizures. Use of magnesium sulfate is the drug therapy of choice for severe preeclampsia and is only used to manage and attempt to prevent progression to eclampsia. Magnesium sulfate therapy does not have as its primary goal a decrease in blood pressure, a decrease in protein in the urine, nor the reversal of edema.

A client in her 38th week of gestation is admitted into the labor and birth unit with painless bleeding from the vagina. The client is diagnosed with placenta previa. When reviewing the client's history, which factor would the nurse identify as a risk factor for placenta previa?

previous cesarean birth A previous cesarean birth is a risk factor for developing placenta previa. This is due to the damage caused to the endometrial tissue. Multiparity, and not a primigravida status, predisposes to placenta previa. Maternal age over 35 years, and not just more than 30 years, is considered another risk factor. Placenta previa is more common among those living in high altitudes not among those living in coastal areas.

A pregnant woman has arrived to the office reporting vaginal bleeding. Which finding during the assessment would lead the nurse to suspect an inevitable abortion?

strong abdominal cramping Strong abdominal cramping is associated with an inevitable spontaneous abortion. Slight vaginal bleeding early in pregnancy and a closed cervical os are associated with a threatened abortion. With an inevitable abortion, passage of the products of conception may occur. No fetal tissue is passed with a threatened abortion.

A pregnant woman is diagnosed with abruptio placentae. When reviewing the woman's medical record, the nurse would expect which finding?

sudden dark, vaginal bleeding The uterus is firm to rigid to the touch with abruptio placentae; it is soft and relaxed with placenta previa. Bleeding associated with abruptio placentae occurs suddenly and is usually dark in color. Bleeding also may not be visible. Bright red vaginal bleeding is associated with placenta previa. Fetal distress or absent fetal heart rate may be noted with abruptio placentae. The woman with abruptio placentae usually experiences constant uterine tenderness on palpation.

A woman in week 16 of her pregnancy calls her primary care provider's office to report that she has experienced abdominal cramping, cervical dilation, vaginal spotting, and the passing of tissue. The nurse instructs the client to bring the passed tissue to the hospital with her. What is the correct rationale for this instruction?

to determine whether gestational trophoblastic disease is present

After a regular prenatal visit, a pregnant client asks the nurse to describe the differences between abruptio placentae and placenta previa. Which statement should the nurse include in the teaching?

"Placenta previa causes painless, bright red bleeding during pregnancy due to an abnormally implanted placenta that is too close to or covers the cervix; abruptio placentae is associated with dark red painful bleeding caused by premature separation of the placenta from the wall of the uterus before the end of labor."

A client in her 20th week of gestation develops HELLP syndrome. What are features of HELLP syndrome? Select all that apply.

-hemolysis -elevated liver enzymes -low platelet count

A primipara at 36 weeks' gestation is being monitored in the prenatal clinic for risk of preeclampsia. Which sign or symptom should the nurse prioritize?

A dipstick value of 2+ for protein.

The nurse is preparing a woman for discharge after a birth and notes the mother's record indicates Rh negative and rubella titer is positive. Which nursing intervention will the nurse prioritize?

Assess the Rh of the baby. The cord blood should be assessed to determine the infant's Rh type. If it is negative, there is no need for any further treatment or concern. However, if it is Rh positive the mother needs to be assessed for possible administration of RhoGAM. The criteria for giving RhoGAM are as follows: The woman must be Rho(D) negative; the woman must not have anti-D antibodies (must not be sensitized); the infant must be Rho(D) positive (fetus cord blood is checked after birth); and a direct Coombs test (a test for antibodies performed on cord blood at delivery) must be weakly reactive or negative. This all needs to be completed within the first 72 hours of birth, so the mother can receive RhoGAM within the proper time frame.

A pregnant patient is diagnosed with placenta previa. Which action should the nurse implement immediately for this patient?

Assess fetal heart sounds with an external monitor. For placenta previa, the nurse should attach external monitoring equipment to record fetal heart sounds and uterine contractions. Internal pressure gauges to measure uterine contractions are contraindicated. A pelvic or rectal examination should never be done with painless bleeding late in pregnancy because any agitation of the cervix when there is a placenta previa might tear the placenta further and initiate massive hemorrhage, which could be fatal to both mother and child. To ensure an adequate blood supply to the patient and fetus, the patient should be placed immediately on bed rest in a side-lying position.

A pregnant patient is diagnosed with placenta previa. Which action should the nurse implement immediately for this patient? Prepare for a vaginal examination to assess the extent of bleeding. Assess uterine contractions by an internal pressure gauge. Help the patient remain ambulatory to reduce bleeding. Assess fetal heart sounds with an external monitor.

Assess fetal heart sounds with an external monitor. Explanation: For placenta previa, the nurse should attach external monitoring equipment to record fetal heart sounds and uterine contractions. Internal pressure gauges to measure uterine contractions are contraindicated. A pelvic or rectal examination should never be done with painless bleeding late in pregnancy because any agitation of the cervix when there is a placenta previa might tear the placenta further and initiate massive hemorrhage, which could be fatal to both mother and child. To ensure an adequate blood supply to the patient and fetus, the patient should be placed immediately on bed rest in a side-lying position.

A pregnant client with multiple gestation arrives at the maternity clinic for a regular antenatal check up. The nurse would be aware that client is at risk for which perinatal complication?

Congenital anomalies

A 25-year-old client at 22 weeks' gestation is noted to have proteinuria and dependent edema on her routine prenatal visit. Which additional assessment should the nurse prioritize and alert the RN or health care provider?

Initial BP 100/70 mm Hg; current BP 140/90 mm Hg

A pregnant woman has been admitted to the hospital due to severe preeclampsia. Which measure will be important for the nurse to include in the care plan?

Institute and maintain seizure precautions. The woman with severe preeclampsia should be maintained on complete bed rest in a dark and quiet room to avoid stimulation. The client is at risk for seizures; therefore, institution and maintenance of seizure precautions should be in place.

A nurse is conducting a presentation for a group of pregnant women about conditions that can occur during pregnancy and that place the woman at high-risk. When discussing blood incompatibilities, which measure would the nurse explain as most effective in preventing isoimmunization during pregnancy?

Rho(D) immune globulin administration to Rh-negative women Rh incompatibility can be prevented with the use of Rho(D) immune globulin. Hemolysis associated with ABO incompatibility is limited to mothers with type O blood and their fetuses with type A or B blood. Amniocentesis would be appropriate for treatment of polyhydramnios, not isoimmunization. Cerclage is a treatment for cervical insufficiency.

A pregnant woman is admitted to the hospital with a diagnosis of placenta previa. Which action would be the priority for this woman on admission?

assessing fetal heart tones by use of an external monitor Not disrupting the placenta is a prime responsibility. An internal monitor, a vaginal examination, and remaining ambulatory could all do this and thus are contraindicated.

The nurse is required to assess a pregnant client who is reporting vaginal bleeding. Which nursing action is the priority?

assessing the amount and color of the bleeding When the woman arrives and is admitted, assessing her vital signs, the amount and color of the bleeding, and current pain rating on a scale of 1 to 10 are the priorities. Assessing the signs of shock, monitoring uterine contractility, and determining the amount of funneling are not priority assessments when a pregnant woman complaining of vaginal bleeding is admitted to the hospital.

A high-risk pregnant client is determined to have gestational hypertension. The nurse suspects that the client has developed severe preeclampsia based on which finding?

blurred vision

Which compound would the nurse have readily available for a client who is receiving magnesium sulfate to treat severe preeclampsia? calcium carbonate potassium chloride calcium gluconate ferrous sulfate

calcium gluconate Explanation: The antidote for magnesium sulfate is calcium gluconate, and this should be readily available in case the woman has signs and symptoms of magnesium toxicity.

A nurse is conducting a refresher program for a group of perinatal nurses. Part of the program involves a discussion of HELLP. The nurse determines that the group needs additional teaching when they identify which aspect as a part of HELLP?

elevated lipoproteins

A woman is admitted with a diagnosis of ectopic pregnancy. For which procedure should the nurse prepare?

immediate surgery Ectopic pregnancy means an embryo has implanted outside the uterus, usually in the fallopian tube. Surgery is usually necessary to remove the growing structure before the tube ruptures or to repair the tube if rupture has occurred already.

A 44-year-old client has lost several pregnancies over the last 10 years. For the past 3 months, she has had fatigue, nausea, and vomiting. She visits the clinic and takes a pregnancy test; the results are positive. Physical examination confirms a uterus enlarged to 13 weeks' gestation; fetal heart tones are heard. Ultrasound reveals that the client is experiencing some bleeding. Considering the client's prenatal history and age, what does the nurse recognize as the greatest risk for the client at this time?

pregnancy loss The client's advanced maternal age (pregnancy in a woman 35 years or older) increases her risk for pregnancy loss. Hypertension, preterm labor, and prematurity are risks as this pregnancy continues. Her greatest risk at 13 weeks' gestation is losing this pregnancy

A woman of 16 weeks' gestation telephones the nurse because she has passed some "berry-like" blood clots and now has continued dark brown vaginal bleeding. Which action would the nurse instruct the woman to do?

"Come to the health facility with any vaginal material passed." This is a typical time in pregnancy for gestational trophoblastic disease to present. Asking the woman to bring any material passed vaginally would be important so it can be assessed for this.

A client experiences a threatened abortion. She is concerned about losing the pregnancy and asks what activity level she should maintain. What is the most appropriate response from the nurse?

"Restrict your physical activity to moderate bedrest." With a threatened abortion, moderate bedrest and supportive care are recommended. Regular physical activity may increase the chances of miscarriage. Strict bedrest is not necessary. Activity restrictions are part of standard medical management.

A 16-year-old client gave birth to a 12 weeks' gestation fetus last week. The client has come to the office for follow-up and while waiting in an examination room notices that on the schedule is written her name and "follow-up of spontaneous abortion." The client is upset about what is written on the schedule. How can the nurse best explain this terminology?

"Spontaneous abortion is a more specific term used to describe a spontaneous miscarriage, which is a loss of pregnancy before 20 weeks. This term does not imply that you did anything to affect the pregnancy." Abortion is a medical term for any interruption of a pregnancy before a fetus is viable, but it is better to speak of these early pregnancy losses as spontaneous abortions to avoid confusion with intentional terminations of pregnancies. The other responses are correct, but they do not provide the client with the most complete and reassuring answer.

During pregnancy a woman's blood volume increases to accommodate the growing fetus to the point that vital signs may remain within normal range without showing signs of shock until the woman has lost what percentage of her blood volume?

40%

A nurse in the maternity triage unit is caring for a client with a suspected ectopic pregnancy. Which nursing intervention should the nurse perform first? Administer oxygen to the client. Provide emotional support to the client and significant other. Obtain a surgical consent from the client. Assess the client's vital signs.

Assess the client's vital signs. Explanation: A suspected ectopic pregnancy can put the client at risk for hypovolemic shock. The assessment of vital signs should be performed first, followed by any procedures to maintain the ABCs. Providing emotional support would also occur, as would obtaining a surgical consent, if needed, but these are not first steps.

A nurse is caring for a young woman who is in her 10th week of gestation. She comes into the clinic reporting vaginal bleeding. Which assessment finding best correlates with a diagnosis of hydatidiform mole?

Dark red, "clumpy" vaginal discharge

The nurse is caring for a pregnant client with fallopian tube rupture. Which intervention is the priority for this client?

Monitor the client's vital signs and bleeding. A nurse should closely monitor the client's vital signs and bleeding (peritoneal or vaginal) to identify hypovolemic shock that may occur with tubal rupture. Beta-hCG level is monitored to diagnose an ectopic pregnancy or impending abortion. Monitoring the mass with transvaginal ultrasound and determining the size of the mass are done for diagnosing an ectopic pregnancy. Monitoring the FHR does not help to identify hypovolemic shock.

A nurse is caring for a client undergoing treatment for ectopic pregnancy. Which symptom is observed in a client if rupture or hemorrhaging occurs before the ectopic pregnancy is successfully treated?

Phrenic nerve irritation

A pregnant patient is developing HELLP syndrome. During labor, which order should the nurse question?

Prepare for epidural anesthesia. In the HELLP syndrome, patients develop low platelet counts. With a low platelet count, injections such as epidural anesthesia are contraindicated. This is the order that the nurse should question. The patient's urine output should be assessed every hour because renal failure is a complication of this syndrome. Positioning on the left side during labor will help blood flow to the uterus. Assessing blood pressure every 15 minutes is appropriate for the patient with this syndrome.

A woman and her partner present at her first antenatal obstetrics appointment. She is 6 weeks pregnant. Her blood tests show that she is Rh negative, although she has not previously known this. She has no other children but suspects that she miscarried early in a pregnancy 2 years before. She felt fine afterward, so she never received medical attention for that suspected episode. Her partner reports that he is also Rh negative. What action will the nurse be likely to take?

The nurse will perform all normal procedures and follow-up tasks because the Rh status of the client and the child are not a concern at this time. If the woman is Rh negative and her partner is Rh negative, the fetus will also be Rh negative, and the woman will not require treatment with Rho(D) immune globulin. Therefore, the remaining options are incorrect.

A pregnant client with multiple gestation arrives at the maternity clinic for a regular antenatal check up. The nurse would be aware that client is at risk for which perinatal complication?

congenital anomalies Multiple gestation involves two or more fetuses. The perinatal complications associated with multiple pregnancy include preterm birth, maternal hypertension and congenital anomalies. Fetal nonimmune hydrops occurs in the infection of pregnant clients with parvovirus. Postterm birth, maternal hypotension, and fetal nonimmune hydrops are not seen as complications of multiple pregnancy.

A woman is being closely monitored and treated for severe preeclampsia with magnesium sulfate. Which finding would alert the nurse to the development of magnesium toxicity in this client?

diminished reflexes

What would be the physiologic basis for a placenta previa?

low placental implantation

A 24-year-old client presents in labor. The nurse notes there is an order to administer RhoGAM after the birth of her infant. When asked by the client the reason for this injection, which reason should the nurse point out?

prevent maternal D antibody formation.

A pregnant client has an Rh-negative blood type. Following the birth of the client's infant, the nurse administers her Rho(D) immune globulin. The purpose of this is to:

prevent maternal D antibody formation.

A client for has an Rh-negative blood type. Following the birth of the client's infant, the nurse administers her Rho(D)immune globulin. The purpose of this is to:

prevent maternal D antibody formation. Because Rho(D) immune globulin contains passive antibodies, the solution will prevent the woman from forming long-lasting antibodies.

After teaching a woman who has had an evacuation for gestational trophoblastic disease (hydatidiform mole or molar pregnancy) about her condition, which statement indicates that the nurse's teaching was successful? "I will be sure to avoid getting pregnant for at least 1 year." "My blood pressure will continue to be increased for about 6 more months." "My intake of iron will have to be closely monitored for 6 months." "I won't use my birth control pills for at least a year or two."

"I will be sure to avoid getting pregnant for at least 1 year." Explanation: After evacuation of trophoblastic tissue (hydatiform mole), long-term follow-up is necessary to make sure any remaining trophoblastic tissue does not become malignant. Serial hCG levels are monitored closely for 1 year, and the client is urged to avoid pregnancy for 1 year because it can interfere with the monitoring of hCG levels. Iron intake and blood pressure are not important aspects of follow up after evacuation of a hydatiform mole. Use of a reliable contraceptive is strongly recommended so that pregnancy is avoided.

A pregnant client at 32 weeks' gestation calls the clinic and informs the nurse that she thinks her membranes are leaking. She states that some clear fluid has run down her leg. What is the best response by the nurse?

"It is best for you to visit a hospital immediately. They can use a nitrazine strip to determine if it is amniotic fluid.

After a regular prenatal visit, a pregnant client asks the nurse to describe the differences between abruptio placenta and placenta previa. Which statement should the nurse include in the teaching?

"Placenta previa causes painless, bright red bleeding during pregnancy due to an abnormally implanted placenta that is too close to or covers the cervix; abruptio placenta is associated with dark red painful bleeding caused by premature separation of the placenta from the wall of the uterus before the end of labor." Placenta previa is a condition of pregnancy in which the placenta is implanted abnormally in the lower part of the uterus and is the most common cause of painless bright red bleeding in the third trimester. Abruptio placenta is the premature separation of a normally implanted placenta that pulls away from the wall of the uterus either during pregnancy or before the end of labor.

A client with a history of cervical insufficiency is seen for reports of pink-tinged discharge and pelvic pressure. The primary care provider decides to perform a cervical cerclage. The nurse teaches the client about the procedure. Which client response indicates that the teaching has been effective? "The cervix is glued shut so no amniotic fluid can escape." "Purse-string sutures are placed in the cervix to prevent it from dilating." "Staples are put in the cervix to prevent it from dilating." "A cervical cap is placed so no amniotic fluid can escape."

"Purse-string sutures are placed in the cervix to prevent it from dilating." Explanation: The cerclage, or purse string suture is inserted into the cervix to prevent preterm cervical dilatation and pregnancy loss. Staples, glue, or a cervical cap will not prevent the cervix from dilating.

A 16-year-old client gave birth to a 12 weeks' gestation fetus last week. The client has come to the office for follow-up and while waiting in an examination room notices that on the schedule is written her name and "follow-up of spontaneous abortion." The client is upset about what is written on the schedule. How can the nurse best explain this terminology?

"Spontaneous abortion is a more specific term used to describe a spontaneous miscarriage, which is a loss of pregnancy before 20 weeks. This term does not imply that you did anything to affect the pregnancy." Abortion is a medical term for any interruption of a pregnancy before a fetus is viable, but it is better to speak of these early pregnancy losses as spontaneous abortions to avoid confusion with intentional terminations of pregnancies. The other responses are correct, but they do not provide the client with the most complete and reassuring answer.

A 28-year-old client and her current partner present for the first antenatal OB appointment. The client has no children but does question a possible miscarriage 2 years ago; however, she never sought medical attention because she felt fine. Labs reveal both client and partner are Rh negative. Which action should the nurse prioritize?

Assess client for anti-D antibodies The client should be checked for sensitization to Rh-positive blood. It is unknown if the client did have a miscarriage earlier, and if so, what the blood type was of that fetus. The risk is high for the current fetus to be affected with hemolytic disease and this can be easily ruled out by assessing the mother for sensitization. If this screening is negative, then no further testing is required. If the father were Rh positive, then the mother be given RhoGAM to prevent the woman from developing antibodies to the Rho(D) factor. However, if it is positive, the health care provider may order an amniocentesis to evaluate the fetus for hemolytic disease so proper treatment and monitoring may be given. It is too early to perform a direct Coombs test. It would be improper to ignore the potential of serious complications and simply continue with routine tasks and procedures at this time.

A pregnant patient is diagnosed with placenta previa. Which action should the nurse implement immediately for this patient?

Assess fetal heart sounds with an external monitor.

A woman in her 20s has experienced a miscarriage at 10 weeks' gestation and asks the nurse at the hospital what went wrong. She is concerned that she did something that caused her to lose her baby. The nurse can reassure the woman by explaining that the most common cause of spontaneous miscarriage in the first trimester is related to which factor?

Chromosomal defects in the fetus

A nurse is caring for a pregnant client with eclamptic seizure. Which is a characteristic of eclampsia?

Coma occurs after seizure. The nurse should know that coma usually follows an eclamptic seizure. Muscle rigidity occurs after facial twitching. Respirations do not become rapid during the seizure; they cease. Coma usually follows the seizure activity, with respiration resuming.

What special interventions would the nurse implement in a client who is carrying twin fetuses? Schedule non-stress tests (NST) starting at 16 weeks. Remind the client to monitor her intake since she does not need any more food for a multiple pregnancy than she would ingest for a singleton pregnancy. Assist the physician on doing uterine ultrasounds every 2 weeks to monitor fetal size and placental information. Demonstrate to the client how to perform fetal movement counts after 32 weeks.

Demonstrate to the client how to perform fetal movement counts after 32 weeks. Explanation: A woman carrying a multiple gestation needs to keep up with how her fetuses are doing, and an excellent way to do that is by doing fetal movement counts, or "kick counts" as they are sometimes called. This starts at around 32 weeks' gestation for an uncomplicated pregnancy and continues until delivery. Weekly or bi-weekly NSTs begin after 32 weeks. Obstetrical ultrasounds are done every 4 to 6 weeks after confirmation of a multiple fetal pregnancy. The client needs to increase her intake, along with her iron and folic acid intake, to provide adequate nutrition for both fetuses.

A 28-year-old client with a history of endometriosis presents to the emergency department with severe abdominal pain and nausea and vomiting. The client also reports her periods are irregular with the last one being 2 months ago. The nurse prepares to assess for which possible cause for this client's complaints?

Ectopic pregnancy

A patient is admitted with a diagnosis of ectopic pregnancy. For what should the nurse anticipate preparing the patient? Immediate surgery Intravenous administration of a tocolytic Bed rest for the next 4 weeks Internal uterine monitoring

Immediate surgery Explanation: An ectopic pregnancy is one in which implantation occurred outside the uterine cavity, usually within the fallopian tube. As the embryo grows, the fallopian tube can rupture. The therapy for ruptured ectopic pregnancy is laparoscopy to ligate the bleeding vessels and to remove or repair the damaged fallopian tube. There is no reason to begin uterine monitoring. The patient does not need to be on bed rest for 4 weeks. A tocolytic is not needed because the patient is not in labor.

The nurse is caring for an Rh-negative nonimmunized client at 14 weeks' gestation. What information would the nurse provide to the client?

Obtain Rho(D) immune globulin at 28 weeks' gestation. The current recommendation is that every Rh-negative nonimmunized woman receives Rho(D) immune globulin at 28 weeks' gestation and again within 72 hours after giving birth. Consuming a well-balanced nutritional diet and avoiding sexual activity until after 28 weeks will not help to prevent complications of blood incompatibility. Transvaginal ultrasound helps to validate the position of the placenta and will not help to prevent complications of blood incompatibility.

A pregnant patient is developing HELLP syndrome. During labor, which order should the nurse question? Assess blood pressure every 15 minutes. Prepare for epidural anesthesia. Position on the left side during labor. Assess urine output every hour.

Prepare for epidural anesthesia. Explanation: In the HELLP syndrome, patients develop low platelet counts. With a low platelet count, injections such as epidural anesthesia are contraindicated. This is the order that the nurse should question. The patient's urine output should be assessed every hour because renal failure is a complication of this syndrome. Positioning on the left side during labor will help blood flow to the uterus. Assessing blood pressure every 15 minutes is appropriate for the patient with this syndrome.

The nurse is caring for a client with preeclampsia and understands the need to auscultate this client's lung sounds every 2 hours. Why would the nurse do this?

Pulmonary edema

A female client presents to the emergency department reporting she recently had a positive home pregnancy test but is now suspected of having an ectopic pregnancy. Which assessment should the nures prioritize?

Shoulder pain

A pregnant patient with a history of premature cervical dilatation undergoes cervical cerclage. Which outcome indicates that this procedure has been successful? The client delivers a full-term fetus at 39 weeks' gestation. The client experiences minimal vaginal bleeding throughout the pregnancy. The client has reduced shortness of breath and abdominal pain during the pregnancy. The client's membranes spontaneously rupture at week 30 of gestation.

The client delivers a full-term fetus at 39 weeks' gestation. Explanation: Premature cervical dilatation is when the cervix dilates prematurely and cannot retain a fetus until term. After the loss of one child because of premature cervical dilatation, a surgical operation termed cervical cerclage can be performed to prevent this from happening in a second pregnancy. This procedure is the use of purse-string sutures placed in the cervix to strengthen the cervix and prevent it from dilating until the end of pregnancy. Evidence that this procedure is effective would be the client delivering a full-term fetus at 39 weeks' gestation. Spontaneous rupture of the membranes could indicate that the procedure was not successful. Vaginal bleeding could indicate another health problem or that the procedure was not successful. This procedure does not impact the patient's respirations or amount of abdominal pain while pregnant. These manifestations could indicate another health problem with the pregnancy.

A pregnant patient with a history of premature cervical dilatation undergoes cervical cerclage. Which outcome indicates that this procedure has been successful?

The client delivers a full-term fetus at 39 weeks' gestation. Premature cervical dilatation is when the cervix dilates prematurely and cannot retain a fetus until term. After the loss of one child because of premature cervical dilatation, a surgical operation termed cervical cerclage can be performed to prevent this from happening in a second pregnancy. This procedure is the use of purse-string sutures placed in the cervix to strengthen the cervix and prevent it from dilating until the end of pregnancy. Evidence that this procedure is effective would be the client delivering a full-term fetus at 39 weeks' gestation. Spontaneous rupture of the membranes could indicate that the procedure was not successful. Vaginal bleeding could indicate another health problem or that the procedure was not successful. This procedure does not impact the patient's respirations or amount of abdominal pain while pregnant. These manifestations could indicate another health problem with the pregnancy.

The nurse is transcribing messages from the answering service. Which phone message should the nurse return first?

a 35-year-old, 21-week G3P2 client with blood pressure of 160/110 mm Hg, blurred vision, and whose last blood pressure was 143/99 mm Hg and urine dipstick showed a +2 proteinuria The nurse should call the at-risk 35-year-old client first. She is 21 weeks and has symptoms (blurred vision) of preeclampsia. She also had an increase of protein in her urine (2+) and a 15% increase in her BP. The nurse will need more information to determine if the 38-week client may be in the early stages of labor, and if the 31-week client with flank pain has a kidney infection. The client with malaise and rhinitis will need to talk to the nurse last to find out what over-the-counter medication she is able to take.

A nurse is assessing pregnant clients for the risk of placenta previa. Which client faces the greatest risk for this condition?

a client who had a myomectomy to remove fibroids A previous myomectomy to remove fibroids can be associated with the cause of placenta previa. Risk factors also include advanced maternal age (greater than 30 years old). A structurally defective cervix cannot be associated with the cause of placenta previa. However, it can be associated with the cause of cervical insufficiency. Alcohol ingestion is not a risk factor for developing placenta previa but is associated with abruption placenta.

A client is admitted at 22 weeks' gestation with advanced cervical dilatation to 5 centimeters, cervical insufficiency, and a visible amniotic sac at the cervical opening. What is the primary goal for this client at this point?

bed rest to maintain pregnancy as long as possible At 22 weeks' gestation, the fetus is not viable. The woman would be placed on bed rest, total, with every attempt made to halt any further progression of dilatation as long as possible. The nurse would not want this fetus to be born vaginally at this stage of gestation. It is not the nurse's responsibility to notify the client's social support of a possible loss of the pregnancy. It is not appropriate at this time to educate the mother on causes of cervical insufficiency for future pregnancies

A 28-year-old woman presents in the emergency department with severe abdominal pain. She has not had a normal period for 2 months, but she reports that that is not abnormal for her. She has a history of endometriosis. What might the nurse suggest to the primary care provider as a possible cause of the client's abdominal pain?

ectopic pregnancy Ectopic pregnancy can present with severe unilateral abdominal pain. Given the history of the client and the amount of pain, the possibility of ectopic pregnancy needs to be considered. A healthy pregnancy would not present with severe abdominal pain unless the client were term and she was in labor. With a molar pregnancy the woman typically presents between 8 to 16 weeks' gestation reporting painless (usually) brown to bright red vaginal bleeding. Placenta previa typically presents with painless, bright red bleeding that begins with no warning.

A nurse is conducting a refresher program for a group of perinatal nurses. Part of the program involves a discussion of HELLP. The nurse determines that the group needs additional teaching when they identify which aspect as a part of HELLP?

elevated lipoproteins The acronym HELLP represents hemolysis, elevated liver enzymes, and low platelets. This syndrome is a variant of preeclampsia/eclampsia syndrome that occurs in 10% to 20% of clients whose diseases are labeled as severe.

A nurse suspects that a client is developing HELLP syndrome. The nurse notifies the health care provider based on which finding? elevated liver enzymes elevated platelet count hyperglycemia disseminated intravascular coagulopathy (DIC)

elevated liver enzymes Explanation: HELLP is an acronym for hemolysis, elevated liver enzymes, and low platelets. Hyperglycemia is not a part of this syndrome. HELLP may increase the woman's risk for DIC.

A woman with an incomplete abortion is to receive misoprostol. The woman asks the nurse, "Why am I getting this drug?" The nurse responds to the client, integrating understanding that this drug achieves which effect?

ensures passage of all the products of conception Misoprostol is used to stimulate uterine contractions and evacuate the uterus after an abortion to ensure passage of all the products of conception. Rh (D) immunoglobulin is used to suppress the immune response and prevent isoimmunization.

A nurse is monitoring a client with PROM who is in labor and observes meconium in the amniotic fluid. What does the observation of meconium indicate?

fetal distress related to hypoxia When meconium is present in the amniotic fluid, it typically indicates fetal distress related to hypoxia. Meconium stains the fluid yellow to greenish brown, depending on the amount present. A decreased amount of amniotic fluid reduces the cushioning effect, thereby making cord compression a possibility. A foul odor of amniotic fluid indicates infection. Meconium in the amniotic fluid does not indicate CNS involvement.

A novice nurse asks to be assigned to the least complex antepartum client. Which condition would necessitate the least complex care requirements?

gestational hypertension Hypertensive disorders represent the most common complication of pregnancy. Gestational hypertension is elevated blood pressure without proteinuria, other signs of preeclampsia, or preexisting hypertension. Abruptio placenta (separation of the placenta from the uterine wall), placenta previa (placenta covering the cervical os), and preeclampsia are high-risk, potentially life-threatening conditions for the fetus and mother during labor and birth.

A client has been admitted with abruptio placentae. She has lost 1,200 mL of blood, is normotensive, and ultrasound indicates approximately 30% separation. The nurse documents this as which classification of abruptio placentae?

grade 2 The classifications for abruptio placentae are: grade 1 (mild) - minimal bleeding (less than 500 mL), 10% to 20% separation, tender uterus, no coagulopathy, signs of shock or fetal distress; grade 2 (moderate) - moderate bleeding (1,000 to 1,500 mL), 20% to 50% separation, continuous abdominal pain, mild shock, normal maternal blood pressure, maternal tachycardia; grade 3 (severe) - absent to moderate bleeding (more than 1,500 mL), more than 50% separation, profound shock, dark vaginal bleeding, agonizing abdominal pain, decreased blood pressure, significant tachycardia, and development of disseminated intravascular coagulopathy. There is no grade 4

A 44-year-old client has lost several pregnancies over the last 10 years. For the past 3 months, she has had fatigue, nausea, and vomiting. She visits the clinic and takes a pregnancy test; the results are positive. Physical examination confirms a uterus enlarged to 13 weeks' gestation; fetal heart tones are heard. Ultrasound reveals that the client is experiencing some bleeding. Considering the client's prenatal history and age, what does the nurse recognize as the greatest risk for the client at this time?

pregnancy loss

A woman in week 35 of her pregnancy with severe hydramnios is admitted to the hospital. The nurse recognizes that which concern is greatest regarding this client?

preterm rupture of membranes followed by preterm birth Even with precautions, in most instances of hydramnios, there will be preterm rupture of the membranes because of excessive pressure, followed by preterm birth. The other answers are not as big of concerns as preterm birth, in this situation.

When assessing a woman with an ectopic pregnancy, the nurse would suspect that the tube has ruptured based on which finding?

referred shoulder pain Referred pain to the shoulder area indicates bleeding into the abdomen caused by phrenic nerve irritation when a tubal pregnancy ruptures. Vaginal spotting, nausea, and breast tenderness are typical findings of early pregnancy and an unruptured ectopic pregnancy.

The following hourly assessments are obtained by the nurse on a client with preeclampsia receiving magnesium sulfate: 97.3° F (36.2° C), pulse 88 beats/min, respirations 10 breaths/min, blood pressure 148/110 mm Hg. What other priority physical assessment by the nurse should be implemented to assess for potential toxicity?

reflexes Reflex assessment is part of the standard assessment for clients on magnesium sulfate. The first change when developing magnesium toxicity may be a decrease in reflex activity. The health care provider needs to be notified immediately. A change in lung sounds and oxygen saturation are not indicative of magnesium sulfate toxicity. Hourly blood draws to gain information on the magnesium sulfate level are not indicated.

A client with preeclampsia is receiving magnesium sulfate. Which nursing assessment should be ongoing while the medication is being administered?

respiratory rate The level of magnesium in therapeutic range is 4 to 8 mg/dL. If magnesium toxicity occurs, one sign in the client will be a decrease in the respiratory rate and a potential respiratory arrest. Respiratory rate will be monitored when on this medication. The client's hemoglobin and ability to sleep are not factors for ongoing assessments for the client on magnesium sulfate. Urinary output is measured hourly on the preeclamptic client receiving magnesium sulfate, but urine protein is not an ongoing assessment

A woman in week 16 of her pregnancy calls her primary care provider's office to report that she has experienced abdominal cramping, cervical dilation, vaginal spotting, and the passing of tissue. The nurse instructs the client to bring the passed tissue to the hospital with her. What is the correct rationale for this instruction?

to determine whether gestational trophoblastic disease is present Gestational trophoblastic disease is abnormal proliferation and then degeneration of the trophoblastic villi. The embryo fails to develop beyond a primitive start. Abnormal trophoblast cells must be identified because they are associated with choriocarcinoma, a rapidly metastasizing malignancy. This is why it is important for any woman who begins to miscarry at home to bring any clots or tissue passed to the hospital with her. The presence of clear fluid-filled cysts changes the diagnosis from a simple miscarriage to gestational trophoblastic disease. The client is not instructed to bring in passed tissue to determine whether infection is present or the fetus is viable or to determine the stage of development of the fetus.


Kaugnay na mga set ng pag-aaral

Chapter 49: Hepatic D/O Course Point

View Set

Immunization Self-Study Assessment

View Set

Petrus Christus A Goldsmith in His Shop

View Set

PrepU - Chapter 22: Assessing Peripheral Vascular System

View Set

Ultrasound Final Combined with class questions

View Set

Man 4330 Rockmore Exam 2 Compensation Administration UCF

View Set